4 ATTENT. METEO. EXAM. PREP.

Description

4 ATTENT. METEO. EXAM. PREP.
tsampikos pompou
Quiz by tsampikos pompou, updated more than 1 year ago
tsampikos pompou
Created by tsampikos pompou almost 8 years ago
73
5

Resource summary

Question 1

Question
304 When a wind changes direction from Southerly to South-Westerly, it is said to have:
Answer
  • O veered in either hemisphere.
  • O veered in the Northern hemisphere and backed in the Southern hemi¬sphere.
  • O backed in the Northern hemisphere and veered in the Southern hemi¬sphere.
  • O backed in either hemisphere

Question 2

Question
305 A typical warm front extends horizontally for approximately:
Answer
  • O 600 miles.
  • O 200 miles
  • O 100 miles.
  • O 50 miles

Question 3

Question
306 Following the passage of a typical cold front, an observer at the surface would expect to experience which of the following conditions?
Answer
  • O (Pressure)> steady rise. (Clouds)>SCT/BKN Cu. (Visibility)> generally good
  • O (Pressure)> fall then rise. (Clouds)>BKNCu/Scu. (Visibility)> poor with fog
  • O (Pressure)>falling rapidly. (Clouds)>OVC NS/St. (Visibility)>very poor
  • O (Pressure)> rising rapidly. (Clouds)>OVC Ci/Cc. (Visibility)>Poor

Question 4

Question
307 Mist and fog occur when small water droplets are suspended in the air, hence reducing visibility. What is the difference between mist and fog?
Answer
  • O Fog is defined as having a visibility less than 1 kilometre
  • O Fog has a slightly lower Relative Humidity than mist.
  • O Mist is defined as having a visibility less than 1 kilometre.
  • O Fog is defined as having a visibility less than 1.5 kilometre

Question 5

Question
308 Over land in mid latitudes, which statement concerning the diurnal variation oftemperature is most appropriate?
Answer
  • O Maximum temperature occurs at around 1400-1500 LMT, with the minimum temperature occurring just before dawn.
  • O Maximum temperature occurs at around 1400-1500 UTC, with the minimum temperature occurring just before dawn.
  • O Maximum temperature occurs around midday, with the minimum temperature occurring just after sunset.
  • O It depends on the longitude

Question 6

Question
312 Cloud described as 'a transparent whitish veil of fibrous or smooth appearance totally or partly covering the sky producing a halo phenomenon', indicates:
Answer
  • O cirrostratus.
  • O altostratus.
  • O castellanus.
  • O cumulonimbus

Question 7

Question
313 As a parcel of air rises, it cools by the process of:
Answer
  • O adiabatic expansion.
  • O expulsion of long-wave radiation.
  • O conductive mixing.
  • O freezing

Question 8

Question
314 The passage of a cold front means that the pressure:
Answer
  • O rises
  • O falls
  • O remains steady
  • O depends on humidity

Question 9

Question
318 When a wind changes direction from northly to north-westerly, it is said to have:
Answer
  • O backed in either hemisphere
  • O backed in the northern hemisphere and veered in the southern hemisphere.
  • O veered in either hemisphere.
  • O veered in the northern hemisphere and backed in the southern hemisphere.

Question 10

Question
319 You depart from aerodrome of Kastelorizo (LGKJ), (situated 474 feet above mean sea level (AMSL) with a QNH of 1011 hpa set on the altimeter subscale. The flight is conducted towards an area of low pressure and you fail to update the subscale setting (due to being non-radio). You anticipate a visual landing at the aero¬drome οf Kasteli(LGTL). What will the altimeter read when the aircraft lands at the destination aerodrome, which has an elevation of 1180 feet AMSL, if the QNH is 994 hpa?
Answer
  • O 1690
  • O 1750
  • O 950
  • O 1197

Question 11

Question
320 Density will be low when the:
Answer
  • O pressure is low and temperature is high.
  • O pressure is low and temperature is low.
  • O pressure is high and temperature is high.
  • O pressure is high and temperature is low

Question 12

Question
327 The determined surface temperature is +13°C and at 4000 feet it is +6°C. The state of the atmosphere may be described as:
Answer
  • O conditionally unstable.
  • O unstable.
  • O stable.
  • O neutrally stable

Question 13

Question
330 A powerful wind over a mountain range which causes rotor cloud and strong downdraughts, may be marked by:
Answer
  • O lenticularis.
  • O stratus.
  • O cumulus.
  • O cumulonimbus

Question 14

Question
331 Given a surface temperature of +22°C and a dewpoint of +10°C, at approxi¬mately what height is the base of any cumuliform cloud likely to be?
Answer
  • O 5000 feet.
  • O 3500 feet.
  • O 2500 feet.
  • O 3000 feet

Question 15

Question
334 What does the following illustration depict? (See Fig.33)
Answer
  • O A cold occlusion.
  • O A warm occlusion.
  • O A warm front
  • O A cold front

Question 16

Question
337 As a warm front passes, the weather gradually becomes:
Answer
  • O moderate continuous rain changing to intermittent slight drizzle
  • O moderate continuous rain changing to rain showers.
  • O slight showers changing to moderate continuous drizzle.
  • O none of those

Question 17

Question
340 When flying towards an area of low pressure at a constant indicated altitude the true altitude is:
Answer
  • O lower than indicated.
  • O the same as indicated.
  • O higher than indicated.
  • O none of those

Question 18

Question
341 A thin, white patch, sheet or layer of cloud without shading, composed of very small elements in the form of grains or ripples joined together or separate and regularly arranged.' To what type of cloud does this statement refer?
Answer
  • O Cirrocumulus.
  • O Stratocumulus.
  • O Cumulus.
  • O Lenticularis

Question 19

Question
344 If the 2000 feet wind is 240035 knots, the most likely surface wind is:
Answer
  • O 220/020 knots.
  • O 270/040 knots.
  • O 220/040 knots.
  • O 250/040 knots.

Question 20

Question
345 An aircraft is flying on a VFR cross-country. The altimeter is indicating 2500 feet with the current Regional QNH set on the subscale. It is heading towards an area of low pressure. If the pilot fails to reset the subscale to the new Regional QNH, the aircraft will:
Answer
  • O gradually descend.
  • O gradually climb.
  • O maintain the original altitude.
  • O none of those

Question 21

Question
348 Lumpy or heaped cloud belongs to which family?
Answer
  • O Cumuliform.
  • O Stratiform.
  • O Cirriform.
  • O Lenticularis

Question 22

Question
349 . An inversion means that the temperature …………………………. as height is gained
Answer
  • O increases
  • O decreases
  • O remains
  • O decreases below freezing

Question 23

Question
353 Water vapour changes to ice. This process is called…………………………….. and …………...………….latent heat.
Answer
  • O deposition / releases
  • O condensation / absorbs
  • O sublimation / releases
  • O evaporation/ absorbs

Question 24

Question
354 Which of the following c onditions are the most suitable for the formation ofradiation fog?
Answer
  • O (clouds)>cloudless (air state)> moist (winds)> light
  • O (clouds)>cloudless (air state)>moist (winds)>nil
  • O (clouds)>cloudy (air state)>dry and cool (winds)>moderate
  • O (clouds)>cloudy (air state)>moist (winds)>strong

Question 25

Question
355 Which weather conditions should be expected beneath a low-level temperature inversion layer when the relative humidity is high?
Answer
  • O Smooth air, poor visibility, fog, haze, or low clouds.
  • O Light wind shear, poor visibility, haze, and light rain.
  • O Turbulent air, poor visibility, fog, low stratus type clouds, and shower precipitation
  • O Updrafts and cumulus type clouds

Question 26

Question
356 Decode the following METAR for Kavala (LGKV): 301220 LGKV 24015KT 200V280 8000 -RA SCT010 BKN025 OVC080 18/15 Q993 TEMPO 3000 RA BKN008 OVC020=
Answer
  • O Surface wind: mean 240° true, 15 knots; varying between 200° and 280°; minimum visibility 8 km; slight rain; cloud: 3-4 oktas base 1000 feet, 5-7 oktas base 2500 feet, 8 oktas base 8000 feet; temperature +18°C, dewpoint + 15°C; QNH 993 mb; Trend: temporarily, lasting less than hour, or, if recurring, lasting less than half the trend, visibility 3000 metres in rain with 5-7 oktas at base 800 feet, 8 oktas 2000 feet.
  • O Surface wind: 240° true, 15 knots; RVR varying between 200 and 280 metres; minimum visibility 8 km; slight rain; cloud: 1-3 oktas base 1000 feet, 4-7 oktas base 2500 feet, 8 oktas base 8000 feet; temperature +18°C, dewpoint +15°C; QNH 993 mb; Trend: temporarily, lasting less than hour, visibility 3000 metres in rain with 5-7 oktas at base 800 feet, 8 oktas 2000 feet.
  • O Surface wind: mean-240° true, 15 knots; varying between 200° and 280°; minimum visibility 8000 metres in rain; cloud: 1-2 oktas base 1000 feet, 5-7 oktas base 2500 feet, 8 oktas base 800 feet; temperature +18°C, dewpoint +15°C; QNH 993 mb; Trend: temporarily, lasting less than hour, or, if recurring, lasting less than half the trend, visibility 3000 metres in rain with 5-7 oktas at base 800 feet, 8 oktas 2000 feet.
  • O Surface wind: mean-240° true, 15 knots; varying between 200° and 280°; minimum visibility 8000 metres in rain; cloud: 1-2 oktas base 1000 feet, 3-4 oktas base 2500 feet, 8 oktas base 800 feet; temperature +18°C, dewpoint +15°C; QNH 993 mb; Trend: temporarily, lasting less than hour, or, if recurring, lasting less than half the trend, visibility 3000 metres in rain with 5-7 oktas at base 800 feet, 8 oktas 200 feet.

Question 27

Question
358 Maximum temperature occurs at approximately what time and why?
Answer
  • O 14-1500 LMT and occurs when the incoming solar radiation is matched by the outgoing terrestrial radiation.
  • O 1200 UTC and occurs when solar radiation is at maximum.
  • O 1500 LMT and occurs when terrestrial radiation is at maximum.
  • O it depends on the longitude

Question 28

Question
359 If you stand with your back to the wind in the northern hemisphere, you assume that the low pressure is:
Answer
  • O on your left.
  • O in front of you.
  • O on your right.
  • O behind you

Question 29

Question
362 Select the correct following statement concerning high level cloud:
Answer
  • O (base)>20,000 feet (name)>cirriform (precipitation)>none
  • O (base)>20,000 feet (name)>nimbus (precipitation)>continuous rain
  • O (base)>15,000 feet (name)>cumuliform (precipitation)>showers
  • O (base)>8000 feet (name)>stratus (precipitation)>drizzle

Question 30

Question
363 If the surface wind is 240015 knots, what is it expected to be at 2000 feet?
Answer
  • O 270/025 knots.
  • O 280/015 knots.
  • O 2200/25 knots.
  • O 2800/10 knots.
Show full summary Hide full summary

Similar

Circulatory System
bridget.watts97
unit 1 f321 chemistry ocr
methmip
Psychology Exam review
emaw757
How to revise
KimberleyC
Nutrition
manu_maus
mcolby MITOSIS TEST
Melinda Colby
Accounting I - UNIT 1 Fundamentals
Kathleen Keller
Compensation and Benefits PHR Study Guide
Cari Hawthorne
Marketing and Distributing
Shannon Clarke
salesforce Developer 1
sbchowdary
Romeo and Juliet notes
Faizaan Mohmed